Proving Commutativity in Normal Subgroups with Abelian Subgroup Problem

  • Thread starter Thread starter e(ho0n3
  • Start date Start date
  • Tags Tags
    Subgroup
Click For Summary
To prove that the product of elements from two normal subgroups N and M of a group G is commutative, it is essential to show that NM is abelian. Given that N and M intersect only at the identity, the relationship between elements n in N and m in M can be expressed through their commutator, nmn^(-1)m^(-1). Utilizing the normality of both subgroups allows for the transformation of this expression into elements within N and M, leading to the conclusion that nm = mn. The key insight is recognizing the commutator's role in establishing the desired equality. This approach effectively demonstrates the abelian nature of the subgroup product.
e(ho0n3
Messages
1,349
Reaction score
0
Homework Statement
Suppose that N and M are two normal subgroups of G and that N and M share only the identity element. Show that for any n in N and m in M, nm = mn.

The attempt at a solution
I basically have to show that NM is abelian. Since N and M are normal, it follows that

nm = mn_1 = n_1m_1 = m_1n_2 = \cdots,

where n_1,n_2,\ldots \in N and m_1,m_2,\ldots \in M. This is all I know. I don't know how to use the fact that N \cap M = \{e\} or how it plays any role. Any tips?
 
Physics news on Phys.org
Sure. Stare at n*m*n^(-1)*m^(-1) for a little bit. If you still don't see it, I'll give you another hint.
 
Since M is normal, nmn-1 equals some m' in M and since N is normal, mn-1m equals some n' in N. This yield that m'm-1 = nn'. Aha! Then m'm and nn' must be the identity so m' is the inverse of m and n' is the inverse of n and after some rearrangement, we get that nm = mn.

Nice. What made you think of looking at nmn1m-1?
 
I thought you'd get it written in that form. nm=mn is the same thing as nmn^(-1)m^(-1)=e. It even has a name, that expression is called the commutator. I think of commuting, I think of commutator. Just experience, I guess.
 
Question: A clock's minute hand has length 4 and its hour hand has length 3. What is the distance between the tips at the moment when it is increasing most rapidly?(Putnam Exam Question) Answer: Making assumption that both the hands moves at constant angular velocities, the answer is ## \sqrt{7} .## But don't you think this assumption is somewhat doubtful and wrong?

Similar threads

  • · Replies 2 ·
Replies
2
Views
1K
  • · Replies 4 ·
Replies
4
Views
4K
  • · Replies 5 ·
Replies
5
Views
2K
  • · Replies 4 ·
Replies
4
Views
2K
Replies
2
Views
2K
  • · Replies 7 ·
Replies
7
Views
2K
Replies
14
Views
4K
  • · Replies 4 ·
Replies
4
Views
2K
Replies
2
Views
1K
  • · Replies 4 ·
Replies
4
Views
6K